Những câu hỏi liên quan
Tùng Nguyễn
Xem chi tiết
fan FA
Xem chi tiết
tth_new
16 tháng 1 2019 lúc 19:38

Mình có cách này,không chắc lắm:

\(VT=\frac{a}{a\left(a^2+bc+1\right)}+\frac{b}{b\left(b^2+ac+1\right)}+\frac{c}{c\left(c^2+ab+1\right)}\) (làm tắt,bạn tự hiểu nha)

\(=\frac{1}{a^2+bc+1}+\frac{1}{b^2+ac+1}+\frac{1}{c^2+ab+1}\)

\(\le\frac{1}{3}\left(\frac{1}{\sqrt[3]{a}}+\frac{1}{\sqrt[3]{b}}+\frac{1}{\sqrt[3]{c}}\right)\)

\(=\frac{1}{3}\left[\left(1+1+1\right)-\left(\frac{\sqrt[3]{a}-1}{\sqrt[3]{a}}+\frac{\sqrt[3]{b}-1}{\sqrt[3]{b}}+\frac{\sqrt[3]{c}-1}{\sqrt[3]{c}}\right)\right]\)

\(=1-\frac{1}{3}\left(\frac{\sqrt[3]{a}-1}{\sqrt[3]{a}}+\frac{\sqrt[3]{b}-1}{\sqrt[3]{b}}+\frac{\sqrt[3]{c}-1}{\sqrt[3]{c}}\right)\)

Áp dụng BĐT Cô si với biểu thức trong ngoặc:

\(=1-\frac{1}{3}\left(\frac{\sqrt[3]{a}-1}{\sqrt[3]{a}}+\frac{\sqrt[3]{b}-1}{\sqrt[3]{b}}+\frac{\sqrt[3]{c}-1}{\sqrt[3]{c}}\right)\)

\(\le1-\sqrt[3]{\left(\sqrt[3]{a}-1\right)\left(\sqrt[3]{b}-1\right)\left(\sqrt[3]{c-1}\right)}\le1^{\left(đpcm\right)}\)

Dấu "=" xảy ra khi a = b = c = 1

Bình luận (0)
Đen đủi mất cái nik
17 tháng 1 2019 lúc 19:41

Ta c/m bđt sau: 

\(a^3+1\ge a^2+a\)

\(\Leftrightarrow a^3+1-a^2-a\ge0\Leftrightarrow a\left(a^2-1\right)-\left(a^2-1\right)\ge0\Leftrightarrow\left(a-1\right)^2\left(a+1\right)\ge0\)

\(\Rightarrow\frac{a}{a^3+a+1}\le\frac{a}{a^2+2a}=\frac{1}{a+2}\)

\(\Rightarrow\frac{a}{a^3+a+1}+\frac{b}{b^3+b+1}+\frac{c}{c^3+c+1}\le\frac{1}{a+2}+\frac{1}{b+2}+\frac{1}{c+2}\)

Đặt \((a,b,c)\rightarrow(\frac{x}{y},\frac{y}{z},\frac{z}{x})\)

\(\Rightarrow\frac{1}{a+2}+\frac{1}{b+2}+\frac{1}{c+2}=\frac{y}{x+2y}+\frac{z}{y+2z}+\frac{x}{z+2x}=\frac{1}{2}\left(1-\frac{x}{x+2y}+1-\frac{y}{y+2z}+1-\frac{z}{z+2x}\right)=\frac{3}{2}-\frac{1}{2}\left(\frac{x^2}{x^2+2xy}+\frac{y^2}{y^2+2yz}+\frac{z^2}{z^2+2xy}\right)\)\(\le\frac{3}{2}-\frac{1}{2}\left(\frac{\left(x+y+z\right)^2}{x^2+y^2+z^2+2xy+2yz+2zx}\right)=\frac{3}{2}-\frac{1}{2}.\frac{\left(x+y+z\right)^2}{\left(x+y+z\right)^2}=1\)

Dấu bằng xảy ra khi a=b=c=1

Bình luận (0)
tth_new
6 tháng 5 2019 lúc 18:55

Thấy mọe rồi,lúc đó t ngốc quá nên làm nhầm.

Bình luận (0)
Nguyễn Hoàng Tiến
Xem chi tiết
không cần biết
Xem chi tiết
ღ๖ۣۜLinh
20 tháng 2 2020 lúc 10:33

Sửa đề:  Cho a, b, c là các số thực dương thỏa mãn điều kiện abc=1. Chứng minh rằng

\(\frac{1}{ab+b+2}+\frac{1}{bc+c+2}+\frac{1}{ca+a+2}\le\frac{3}{4}\)

Áp dụng bđt Cauchy-Schwarz ta có:

\(\frac{1}{ab+b+2}=\frac{1}{ab+1+b+1}\le\frac{1}{4}\left(\frac{1}{ab+1}+\frac{1}{b+1}\right)\) \(=\frac{1}{4}\left(\frac{abc}{ab\left(1+c\right)}+\frac{1}{b+1}\right)=\frac{1}{4}\left(\frac{c}{1+c}+\frac{1}{b+1}\right)\)

Tương tự \(\frac{1}{bc+c+2}\le\frac{1}{4}\left(\frac{a}{a+1}+\frac{1}{c+1}\right)\)

          \(\frac{1}{ca+a+2}\le\frac{1}{4}\left(\frac{b}{b+1}+\frac{1}{a+1}\right)\)

Cộng từng vế các bđt trên ta được

\(VT\le\frac{1}{4}\left(\frac{a+1}{a+1}+\frac{b+1}{b+1}+\frac{c+1}{c+1}\right)=\frac{3}{4}\)

Vậy bđt được chứng minh

Dấu "=" xảy ra khi a=b=c=1

Bình luận (0)
 Khách vãng lai đã xóa
DOC CO CAU BAI
Xem chi tiết
Trần Nguyễn Ngọc Hưng
Xem chi tiết

Ta đổi chiều bất đẳng thức, khi đó bất đẳng thức cần chứng minh tương đương với:

\(18\left(\frac{a^3}{1+a^3}+\frac{b^3}{1+b^3}+\frac{c^3}{1+c^3}\right)+\left(a+b+c\right)^3\ge54\)

Để ý abc=1 thì \(\frac{a^3}{1+a^3}=\frac{a^3}{abc+a^3}=\frac{a^2}{bc+a^2}\)nên bất đẳng thức trên thành:

\(18\left(\frac{a^2}{bc+a^2}+\frac{b^2}{ca+b^2}+\frac{c^2}{ab+c^2}\right)+\left(a+b+c\right)^3\ge54\)

Lại cũng từ \(abc=1\) ta có \(\left(a+b+c\right)^3\ge27abc=27\), do đó ta sẽ chứng minh được khi ta chỉ ra được:

\(\frac{a^2}{bc+a^2}+\frac{b^2}{ca+b^2}+\frac{c^2}{ab+c^2}\ge\frac{3}{2}\)

Vế trái của đánh giá trên áp dụng bất đẳng thức Bunhiacopxki dạng phân thức. Lúc này ta được:

\(\frac{a^2}{bc+a^2}+\frac{b^2}{ca+b^2}+\frac{c^2}{ab+c^2}\ge\frac{\left(a+b+c\right)^2}{a^2+b^2+c^2+ab+bc+ca}\)

Tuy nhiên để đến khi \(a=b=c=1\) thì:

\(\frac{18\left(a+b+c\right)^2}{a^2+b^2+c^2+ab+bc+ca}=\left(a+b+c\right)^3=27\)

Ta sử dụng bất đẳng thức Cauchy dạng \(x+y\ge2\sqrt{xy}\), khi đó ta được:

\(\frac{18\left(a+b+c\right)^2}{a^2+b^2+c^2+ab+bc+ca}+\left(a+b+c\right)^3\ge\sqrt{\frac{18\left(a+b+c\right)^5}{a^2+b^2+c^2+ab+bc+ca}}\)

Chứng minh sẽ hoàn tất nếu ta chỉ được:

\(\sqrt{\frac{18\left(a+b+c\right)^5}{a^2+b^2+c^2+ab+bc+ca}}\ge54\Leftrightarrow\left(a+b+c\right)^5\ge\frac{81}{2}\left(a^2+b^2+c^2+ab+bc+ca\right)\)

Vậy theo bất đẳng thức Cauchy ta được:

\(\left(a+b+c\right)^6=\left[\left(a^2+b^2+c^2\right)+\left(ab+bc+ca\right)+\left(ab+bc+ca\right)\right]^3\)

\(\ge27\left(a+b+c\right)^2\left(ab+bc+ca\right)^2\ge81abc\left(a^2+b^2+c^2\right)\left(a+b+c\right)\)

\(=81\left(a^2+b^2+c^2\right)\left(a+b+c\right)\)

Khi đó ta được:

\(\left(a+b+c\right)^5\ge81\left(a^2+b^2+c^2\right)\)

Vậy ta cần chỉ ra rằng:

\(2\left(a^2+b^2+c^2\right)\ge a^2+b^2+c^2+ab+bc+ca\)

Vậy bất đẳng thức trên tương đương với \(\left(a-b\right)^2+\left(b-c\right)^2+\left(c-a\right)^2\ge0\), là một bất đẳng thức hiển nhiên đúng.

Vậy bất đẳng thức được chứng minh, dấu đẳng thức xảy ra khi \(a=b=c=1\)

Bình luận (0)
 Khách vãng lai đã xóa
Kiệt Nguyễn
Xem chi tiết
Lê Minh Đức
Xem chi tiết
Kuramajiva
Xem chi tiết
Nguyễn Việt Lâm
30 tháng 12 2020 lúc 21:34

1. Đề thiếu

2. BĐT cần chứng minh tương đương:

\(a^4+b^4+c^4\ge abc\left(a+b+c\right)\)

Ta có:

\(a^4+b^4+c^4\ge\dfrac{1}{3}\left(a^2+b^2+c^2\right)^2\ge\dfrac{1}{3}\left(ab+bc+ca\right)^2\ge\dfrac{1}{3}.3abc\left(a+b+c\right)\) (đpcm)

3.

Ta có:

\(\left(a^6+b^6+1\right)\left(1+1+1\right)\ge\left(a^3+b^3+1\right)^2\)

\(\Rightarrow VT\ge\dfrac{1}{\sqrt{3}}\left(a^3+b^3+1+b^3+c^3+1+c^3+a^3+1\right)\)

\(VT\ge\sqrt{3}+\dfrac{2}{\sqrt{3}}\left(a^3+b^3+c^3\right)\)

Lại có:

\(a^3+b^3+1\ge3ab\) ; \(b^3+c^3+1\ge3bc\) ; \(c^3+a^3+1\ge3ca\)

\(\Rightarrow2\left(a^3+b^3+c^3\right)+3\ge3\left(ab+bc+ca\right)=9\)

\(\Rightarrow a^3+b^3+c^3\ge3\)

\(\Rightarrow VT\ge\sqrt{3}+\dfrac{6}{\sqrt{3}}=3\sqrt{3}\)

Bình luận (1)
Nguyễn Việt Lâm
30 tháng 12 2020 lúc 21:37

4.

Ta có:

\(a^3+1+1\ge3a\) ; \(b^3+1+1\ge3b\) ; \(c^3+1+1\ge3c\)

\(\Rightarrow a^3+b^3+c^3+6\ge3\left(a+b+c\right)=9\)

\(\Rightarrow a^3+b^3+c^3\ge3\)

5.

Ta có:

\(\dfrac{a}{b}+\dfrac{b}{c}\ge2\sqrt{\dfrac{a}{c}}\) ; \(\dfrac{a}{b}+\dfrac{c}{a}\ge2\sqrt{\dfrac{c}{b}}\) ; \(\dfrac{b}{c}+\dfrac{c}{a}\ge2\sqrt{\dfrac{b}{a}}\)

\(\Rightarrow\sqrt{\dfrac{b}{a}}+\sqrt{\dfrac{c}{b}}+\sqrt{\dfrac{a}{c}}\le\dfrac{a}{b}+\dfrac{b}{c}+\dfrac{c}{a}=1\)

Bình luận (0)
Nguyễn Việt Lâm
30 tháng 12 2020 lúc 21:39

Câu 1:

\(VT=1-\dfrac{1}{2}+\dfrac{1}{2}-\dfrac{1}{3}+...+\dfrac{1}{n-1}-\dfrac{1}{n}\)

\(VT=1-\dfrac{1}{n}< 1\) (đpcm)

Bình luận (0)